Download as pdf or txt
Download as pdf or txt
You are on page 1of 24

.

 VISIONIAS
 ™™™Ǥ˜‹•‹‘‹ƒ•Ǥ‹
ANSWERS & EXPLANATION
CSAT 2023 – ABHYAAS TEST – 3 – 4256

1 (d)
Assumption 1 is incorrect. The passage mentions, “In this regard, Waste Heat constitutes a substantial amount of
energy in many core sectors such as iron and steel, cement and power plants.” So, we can assume that a substantial
amount of energy is wasted as heat, but we cannot quantify the exact amount in percentage terms. So, it is an
incorrect assumption.
Assumption 2 is incorrect. The passage mentions, “To support efforts towards sustainable growth of the economy,
the reuse of waste heat generated by industries for electricity generation has tremendous potential.” However, we
cannot assume that it will increase the profitability of industries. Such an energy may be cheaper or more expensive.
Also, the passage is not specific to Indian companies. Therefore, this assumption is incorrect.

2 (a)
Statement 1 is correct. The Waste Heat Recovery method will ensure electricity generation from waste heat that
will reduce the usage of fossil fuels. It is evident from the lines, “One of the prime measures to decarbonize
industries, especially steel and cement is to improve the efficiency of the overall process. ….. Waste Heat Recovery
is a method of recycling and re-using the heat that is escaping the industrial processes after useful work is
accomplished.” So, it is a correct statement.
Statement 2 is correct. The passage mentions, “To support efforts towards sustainable growth of the economy, the
reuse of waste heat generated by industries for electricity generation has tremendous potential.” The passage clearly
says that this method will help in moving towards sustainable growth of the economy. So, it is a correct statement.
Statement 3 is incorrect. The passage does not mention anything about the reduction in equipment size. This
statement is beyond the scope of the passage.
Statement 4 is incorrect. The passage does not mention anything about the reduction in emission of toxic liquid
chemicals due to Waste Heat Recovery. This statement is beyond the scope of the passage.

3 (d)
Statement 1 is incorrect. The passage states, “Officials in the Ministry of Home Affairs told The Indian Express
that the licence was suspended following prima facie inputs regarding the violation of funding norms”. This implies
that the reason for suspension of licence was violation of funding norms. This is rather vague. The passage does not
explicitly mention or imply anything about funds received from any prohibited organisation. There could be other
grounds of violation of these norms. Hence, it is not a correct statement.
Statement 2 is incorrect. According to the passage, “…concerns were expressed in Parliament as early as in 1969”
and that “…the FCRA was enacted during the Emergency in 1976”. So, FCRA was not passed in the same year in
which apprehensions regarding foreign powers pumping money in India were raised in the Parliament. Hence, it is
not a correct statement.
Statement 3 is correct. The passage states, “The law sought to regulate foreign donations to individuals and
associations so that they functioned “in a manner consistent with the values of a sovereign democratic republic””.
It is clearly mentioned that FCRA sought to regulate foreign funding as per the sovereign, democratic ideals. Hence,
it is a correct statement.

1 www.visionias.in ©Vision IAS


.

4 (a)
Statement 1 is correct. The passage states, “We hold ourselves to the highest standards of compliance and are
 confident that we have done nothing wrong. We are committed to working with the authorities to address any
questions they might have.” It is clear from this statement that CPR authorities are willing to cooperate with law
enforcement agencies. Hence, it is a correct statement.
Statement 2 is incorrect. The passage states, “CPR, according to its website, is “recognized as a not-for-profit
society by the Government of India and contributions to the Centre are tax exempt. ……””. Although its
contributions to the Centre are tax exempt, we cannot generalize it to mean that no non-profit organization is required
to pay tax. Hence, it is not a correct statement.

5 (b)
[(4/5) of (4/5) ÷ (4/5)] / [(4/5) ÷ (4/5) of (4/5)]
= [(16/25) ÷ (4/5)] / [(4/5) ÷ (16/25)]
= [(16/25) × (5/4)] / [ (4/5) × (25/16)]
= (4/5) / (5/4)
= 16/25
Hence, option (b) is the correct answer.

6 (b)
Let A = 100

From statement 1:
45% of A + B = 125% of B
Or (45/100) × 100 + B = (125/100) × B
Or 45 + B = 5B/4
Or (5B-4B)/4 = 45
Or B = 45×4
Or B = 180

From statement 2:
20% of A + C = 125% of C
Or (20/100) ×100 + C = (125/100) × C
Or 20 + C = 5C/4
Or (5C-4C)/4 = 20
Or C = 20×4
Or C = 80
Therefore, A = 100, B = 180 and C = 80
So, B > A > C (A is less than B, but greater than C)
Hence, option (b) is the correct answer.

2 www.visionias.in ©Vision IAS


.

7 (b)
Given Series: MEDIUM, NGDIUM, NGGMUM, ?

The pattern of the series is as follows:

Hence, option (b) is correct.

8 (a)
The pattern followed here is:
9 + 6 → (9×9) + (9×6) = 81 + 54 = 135
13 + 7 → (13×13) + (13×7) = 169 + 91 = 260
7 + 12 → (7×7) + (7×12) = 49 + 84 = 133
10 + 11 → (10×10) + (10×11) = 100 + 110 = 210
18 + 15 → (18×18) + (18×15) = 324 + 270 = 594
Hence, option (a) is correct.

9 (b)
Total number of persons = 6
The number of ways in which n persons can be seated around a round table = (n – 1)!
So, number of ways in which 6 persons can be seated around a round table = (6 – 1)! = 5! = 120.

Now, let’s calculate the number of arrangements in which no parent sits adjacent to the son.
Let’s place the son at one of the chairs. Now, two out of the three daughters can be seated on either side of the son.
This can be done in 3C2 × 2! ways.
Now, the remaining three people can be arranged in 3! ways.
So, the total number of ways in which the son does not sit adjacent to either of his parents = 3C2 × 2! × 3! = 3 × 2 ×
6 = 36
So, the total number of ways in which the son sits adjacent to at least one of his parents = 120 – 36 = 84
Hence, option (b) is correct.

10 (b)
Q is happier than P, but not as happy as R or S.
So, R/S > Q > P
R is the happiest. So, the arrangement in decreasing order of happiness is:

3 www.visionias.in ©Vision IAS


.

R>S>Q>P


R is richer than Q, but not richer than P.
So, P > R > Q
S is the richest. So, the arrangement in decreasing order of richness is:
S>P>R>Q

Thus, P is the second richest person and S is the second happiest person.
Hence, option (b) is correct.

11 (d)
Assumption 1 is incorrect. The passage does not mention that freebies are a necessity. Rather the lines “Finally,
when the chickens come home to roost, they will cut down on expenditures such as defence, healthcare, etc., to
finance these profligacies”, show that funding such freebies will compromise defence and healthcare expenditures.
Therefore, the assumption that freebies are a necessity would not be correct as per the passage.
Assumption 2 is incorrect. The tone of the passage is against the practice of doling out freebies by political parties.
It is clearly evident from these phrases: “Parties desperate to win elections are promising everything to everyone,
….. Parties have simply miscalculated the political costs, ….. The other benign explanation is the sheer error…”
So, the question of exploring financial solutions for funding the freebies does not even arise. Therefore, this
assumption is invalid.

12 (c)
Option (a) is incorrect. The passage does not mention the relationship between religion and the afterlife. The
passage discusses the philosophical aspect of the afterlife and the debate around it. So, it cannot be considered the
crux of the passage.
Option (b) is incorrect. The passage is not focusing on the time line of debate on afterlife, but on the relevance of
the question of afterlife. It never says that the concept of afterlife is redundant. So, as per the given information,
it is an incorrect statement.
Option (c) is correct. The passage mentions, “One reason is that the values we have in the here and now have a
bearing on what we may or should hope for in terms of the future. While some philosophers believe that what
happens in the future has enormous consequences for life’s meaning now…” So, due to the perception that the
present and future are intertwined, the topic of the afterlife becomes important. So, this answer option aptly
captures the crux of the passage.
Option (d) is incorrect. The passage talks about two types of philosophers – those who think that future events
have enormous consequences for life’s meaning now; and the others who are focused on the importance of the
present. These are peripheral points, not the crux of the passage, incorporated to lend support to the main argument.
Also, this statement is sermonic in a tone which does not align with the passage.

13 (d)
Assumption 1 is incorrect. The passage mentions, “There are many reasons why there should be ongoing attention
as to whether Plato or Epictetus—or any other philosopher who offers a different account about the reality or
illusion of an afterlife—is right.” But we cannot assume what was the exact position of Plato vis-à-vis the concept
of afterlife. So, it is not the correct assumption.
Assumption 2 is incorrect. As explained above, we cannot assume the exact position of Epictetus vis-à-vis afterlife.
So, it is an incorrect assumption.

4 www.visionias.in ©Vision IAS


.

14 (c)
According to the question,


Aman selected an expensive black sport shoes. So, he must have purchased a pair of shoes of Puma.
Hence, option (c) is the correct answer.

15 (a)
Statement 1:
If the sum of the digits of n is divisible by 3, it means that n is divisible by 3. For example, the number 123 is
divisible by 3. The sum of its digits = 1 + 2 + 3 = 6, which is also divisible by 3. Therefore, statement 1 alone is
sufficient to answer the question.
Statement 2:
If the difference between the last two digits of n is divisible by 3, it does not necessarily mean that n is divisible by
3. For example, the number 25 is not divisible by 3, even though the difference between its last two digits (5 - 2 =
3) is divisible by 3. While the number 30 is divisible by 3, and the difference between its last two digits (3 - 0 = 3)
is also divisible by 3. Therefore, statement 2 alone is not sufficient to answer the question.
Hence, option (a) is the correct answer.

16 (c)
Given Series: A, P, E, S, I, Y, O, H, ?, ?
The series given above consists of two series:
First series: A, E, I, O, ?
In this series, vowels are written in ascending order. So, the next term will be U.

Second series: P, S, Y, H, ?
P+3=S
S+6=Y
Y+9=H
? = H + 12 = T
Thus, missing letters are U and T.
Hence, option (c) is correct.

5 www.visionias.in ©Vision IAS


.

17 (d)
37.5% = 3/8

Let the total number of students in the school be 80x.
So, the number of girls in the school = 80x × 3/8 = 30x
And, the number of boys in the school = 80x - 30x = 50x

80% of the girls and 70% of the boys participated in annual sports of the school.
So, the number of girls who did not participate in annual sports = 30x × (100 – 80)% = 30x × (20/100) = 6x
And the number of boys who did not participate in annual sports = 50x × (100 – 70)% = 50x × (30/100) = 15x
Total number of students who did not participate in annual sports = 6x + 15x = 21x
Required percentage = (21x/80x) × 100 = 26.25%
Hence, option (d) is correct.

18 (b)
M = 32y + 20y
an + bn is always divisible by (a + b) when n is odd.
If y is odd, then 32y + 20y is exactly divisible by (32 + 20), i.e. 52. So, M is exactly divisible by 52.
As M is divisible by 52, it must also be divisible by its factors: 2, 4, 13 and 26.
Though M need not be divisible by 6.
Hence, option (b) is correct.

19 (c)
468/192 = 22×68 / (64×3)
= 2136 / (26×3)
= 2130/3 (in the end we will multiply the answer with 26)

21/3→ Remainder is 2
22/3 →Remainder is 1
23/3 →Remainder is 2 …………… and so on.
So, when power is odd, we get remainder 2.
& when power is even, we get remainder 1.

Therefore, in case of 2130/3 we will get remainder 1.


Required Remainder = 1×26 = 64
Hence, option (c) is the correct answer.

20 (a)
Total number of apples in the box = 15
Number of ways of choosing 4 apples from these 15 apples = 15C4 = 15! / (4! × 11!) = (15×14×13×12) / (4×3×2) =
1365
6 www.visionias.in ©Vision IAS
.

Number of ways of choosing 4 rotten apples from 5 rotten apples = 5C4 = 5! / (4! × 1!) = 5
So, Required probability = 5/1365 = 1/273

Hence, option (a) is correct.

21 (d)
Let diagonal of the square be d units.
Area of square = (1/2) d2
If diagonal of the square is reduced by half, then the new area of the square = (1/2) × (d/2)2 = (1/8) d2
So, reduction in the area of the square = (1/2)d2 – (1/8)d2 = (3/8) d2
The percentage by which the area of that square gets reduced = [{(3/8)d2} / {(1/2)d2}] × 100 = 75%
Hence, option (d) is correct.

22 (d)
Option (a) is incorrect. The passage only says that the rise in technological innovations has corresponded with the
rise in the computational capabilities of computers. It is nowhere mentioned that this increase in computational
capability of computers is due to semiconductors. So, as per the passage, it is an incorrect statement.
Option (b) is incorrect. The passage mentions, “However, its ability to self-implement instructions and carry out
these functions based on its training presents us with unique ethical considerations applicable to the use of AI
systems in various capacities.” The author talks about ethical considerations which could be related to potential
risks. This makes this answer option close, even though the author has not delved into the regulatory aspects. Let’s
put it on hold and consider other options.
Option (c) is incorrect as option (a) is incorrect.
Option (d) is correct. The sentence, “These algorithms are trained on massive datasets, i.e., training datasets,
which provide it with a certain amount of input information and output” clearly reflects the role of advancements
in computational power in the emergence of AI. And the line, “presents us with unique ethical considerations
applicable to the use of AI systems in various capacities” highlights the concerns associated with AI. So, this
answer option aptly captures the critical message of the passage.

23 (b)
Inference 1 is incorrect. The passage mentions “Many argue that rapid technological change will raise
agricultural yields so much by late this century that any reduction caused by global warming would easily be more
than offset. But technological change is a false panacea for several reasons.” These lines clearly show that in the
future technology may not solve the issue of reduced yield because of global warming. So, the given inference is
not correct.
Inference 2 is correct. This answer option is close. The passage mentions, “It also seems quite likely that a sizable
share of land will be shifted to the production of biomass for ethanol fuel.” This line reflects that in future cropping
land could be diverted to meet fuel requirements. This might adversely affect food security. It won’t be incorrect to
say that ethanol production could probably become a roadblock for food security in future. Let’s keep this answer
option on hold and consider the next option.
Inference 3 is incorrect. The passage mentions, “Although rising agricultural prices might provide incentives that
would slow or reverse this decline, such a response is not assured.” This line shows that rising prices might provide
incentives which can slow down the reduction of yield, but that is not certain. So, as per the information given in
the passage, this inference is not correct.
Since, statements 1 and 3 are incorrect, options (a), (c) and (d) can be ruled out. Option (b) is the most appropriate
answer.

7 www.visionias.in ©Vision IAS


.

24 (b)
Option (a) is incorrect. The passage mentions, “But technological change is a false panacea for several reasons.”
 Technology is not the focus of the passage. Author is drawing attention towards the precarious balance between
supply and demand when it comes to food, and various challenges that may further deteriorate this balance. Hence,
this is not the best message of the author.
Option (b) is correct. The passage mentions “Global food demand is expected to approximately triple by the 2080s
because of the higher world population and higher incomes. It also seems quite likely that a sizable share of land
will be shifted to the production of biomass for ethanol fuel. As a result, there is a rather precarious balance
between supply and demand, which would be seriously worsened by a major adverse shock from global warming.”
These lines show that food and fuel demand is increasing, and global warming is posing a challenge. So, there is a
need to come up with some solutions to address the given challenges. Therefore, this statement best reflects the
message of the author.
Option (c) is incorrect. The given statement is not correct because it only partially covers the concerns of the
author. The author has focused on both supply and demand sides while talking about the precarious balance between
food supply and demand. In the lines, “Second, even if there is no further slowdown, there is likely to be a close
race between rising food demand and rising output. Global food demand is expected to approximately triple by the
2080s because of the higher world population and higher incomes.” he talks about the challenges on the demand
side too. Also, the issue of malnutrition in developing countries has not been discussed in the passage. Therefore,
this statement is not correct.
Option (d) is incorrect. The given statement is not correct, because the context of organic farming and population
control has not been covered in the passage. So, this cannot be the best message of the author.

25 (c)
The person spent 9 hours in the journey, but gained 1 day, i.e. 24 hours.
Hence, he will reach the destination on 5 P.M. of 1st May + (9 – 24) hours, i.e. 2 A.M. of 1st May.
So, option (c) is the correct answer.

26 (b)
Some of the possible Venn diagrams have been shown below:

We can conclude that “No capital is a city” is not definitely true.


Also, “Some cities are capitals” is not definitely true.
But one of these conclusions must be true.
Hence, option (b) is correct.

27 (d)
Average of the number of different flowers used by the event management company in the month of March = (96 +
80 + 118)/3 = 294/3 = 98
Hence, option (d) is correct.

8 www.visionias.in ©Vision IAS


.

28 (d)
Total number of roses used in January = 72

Total number of lilies used in January = 84
Required ratio = 72:84 = 6:7
Hence, option (d) is correct.

29 (c)
Let the number of each type of article purchased by the man be x.
Then according to the question,
52x + 78x + 108x = 1190
Or 238x = 1190
Or x = 1190/238
Or x = 5
Therefore, the total number of articles purchased by the man = 5 × 3 = 15 articles
Hence, option (c) is the correct answer.

30 (d)
(+) sign → Male
(-) sign → Female

Option (a):

Here, C is female. So, option (a) is false.

Option (b):

Here, C is the uncle of M. So, option (b) is also false.

Option (c):

Here, gender of C is not known. So option (c) is also false.

9 www.visionias.in ©Vision IAS


.

Option (d):

Here, C is the son of M.


Hence, option (d) is the correct answer.

31 (a)
As per the information given in the question, we can conclude that:
Vibha > Teena = Meena ≥ Priya < Hema

VIbha is taller than Priya is definitely true. But there is no direct relation between the heights of Teena and Hema.
Hence, only option (a) is correct.

32 (d)
Given sequence: A E % 5 Y B R & # P Y W @ 2 1 $ 3 * K L M V

After rewriting the given sequence as per the condition given in the question, we get the following resultant
sequence:
AE%5Y$12@WYP#&RB3*KLMV

Now, the fourth element to the left of fifteenth element from the left is Y.
Hence, option (d) is the correct answer.

33 (c)
344 = 86 × 4
1032 = 86 × 12
1806 = 86 × 21
So, HCF of 344, 1032 and 1806 = 86
So, Sumit’s daily expense = 2 × 86 = Rs. 172
The amount invested per day = 172 × (40/100) = Rs. 68.8
Hence, option (c) is the correct answer.

Explanation for questions 34 to 36:


Only two employees have salary less than C. The number of employees having salary higher than C is less than the
number of employees having salary higher than B. So, two cases are possible.
Case 1: - > - > - > C > B > -
Case 2: - > - > - > C > - > B

 10 www.visionias.in ©Vision IAS


.

The number of employees having salary higher than B is equal to the number of employees having salary less than
E. So, we get:
 Case 1: - > E > - > C > B > -
Case 2: E > - > - > C > - > B

Now, F earns a salary of Rs. 15000, which is not higher than the salary of either D or A. So, we get:
Case 1: D/A > E > A/D > C > B > F
Case 2: E > D/A > A/D > C > F > B

A or D do not earn the highest salary. So, we can eliminate Case 1.


The number of employees having salary higher than A is an even number. So, we get:
E>D>A>C>F>B

34 (c)

35 (c)
It is given that salary of F is Rs. 15000, So B’s salary must be less than Rs. 15000, while C’s salary must be more
than Rs. 15000.
So, the statement “Salary of B can be Rs. 16000” cannot be true under any circumstances.

36 (d)

37 (d)
Statement 1 is incorrect. The passage mentions, “Raman discovered that when a stream of light passes through a
liquid, a fraction of the light scattered by the liquid is of a different colour”. It only talks about light in general, and
does not specifically mention the ultraviolet spectrum. Hence, it is not a correct statement.
Statement 2 is incorrect. The passage states, “…when a stream of light passes through a liquid, a fraction of the
light scattered by the liquid is of a different colour”. It does not mention whether the scattering is complete or
partial. Hence, it is not a correct statement.
Statement 3 is incorrect. The passage says, “One of the things that scientists look at when light is scattered is if
the particle it interacts with is able to change its energy”. It implies that there is a change in the energy of the light
– it could be increase or decrease in energy. Hence, it would be incorrect to talk about decrease only.
Statement 4 is correct. According to the passage “The Raman Effect is when the change in the energy of the light
is affected by the vibrations of the molecule or material under observation, leading to a change in its wavelength”.
This implies a change in wavelength of light when it is affected by molecular vibrations. Hence, it is a correct
statement.

38 (d)
Statement 1 is incorrect. The passage clearly states, “The Raman Effect is when the change in the energy of the
light is affected by the vibrations of the molecule or material under observation”. The change actually happens in
reality, and is affected by vibrations at molecule or material level. Therefore, it would be incorrect to say that change
in energy of light is just a delusion.
Statement 2 is incorrect. The passage only mentions, “The Raman Effect was the discovery which won physicist
Sir CV Raman his Nobel Prize in 1930”. It does not say that he was the first Indian to win a Nobel for science.
Hence, it is not a correct statement.
 11 www.visionias.in ©Vision IAS
.

39 (c)
Statement 1 is correct. The passage mentions, “Traditional fire burning practices or ‘cultural burns’ by
 indigenous Australians can help protect the iconic koala, new research by the University of the Sunshine Coast has
found”. Hence, it is a correct statement.
Statement 2 is incorrect. The passage says, “The ‘cultural burns’ were found to be cooler, lower and slower than
‘hot fires’, according to a statement by the university. Such fire activity encouraged the regeneration of suitable
native plants”. The passage does not mention that it is antithetical (unsuitable) to alien plant species. It only says
that these fires favour certain native plants. Hence, it is not a correct statement.
Statement 3 is correct. The passage states, “The ‘cultural burns’ were found to be cooler, lower and slower than
‘hot fires’, according to a statement by the university” and “The research results showed that there were no negative
impacts on the densities or stress levels of the animals during or after the traditional burning method was used”.
This clearly implies that cultural burns are relatively better than hot fires and have less negative impact on wildlife.
Hence, it is a correct statement.
Statement 4 is correct. The passage states, “The United Nations had taken note of burning practices and techniques
of indigenous peoples around the world as a method to control wildfire incidents in a report last year on increasing
incidences of fires globally”. Hence, cultural burns can be leveraged to control wildfires.

40 (a)
Statement 1 is correct. The passage clearly states, “The ‘cultural burns’ were found to be cooler, lower and slower
than ‘hot fires’, according to a statement by the university. Such fire activity encouraged the regeneration of suitable
native plants”. Hence, it is correct to say that some fire activities may facilitate plant regrowth.
Statement 2 is incorrect. The passage says, “Traditional fire burning practices or ‘cultural burns’ by indigenous
Australians can help protect the iconic koala”. It nowhere mentions that it is found only among the indigenous
people of Australia. There could be other communities also who practice such traditional fire burning. Hence, it is
not a correct statement.

41 (a)
The 3-digit numbers that leave a remainder of 5 when divided by 7 are:
103, 110, 117, ………….., 999
The given terms are in A.P., with a common difference of 7.

nth term, an = a + (n – 1) d
Here, a = 103, an = 999, d = 7

So, 999 = 103 + (n – 1)7


Or (n – 1)7 = 999 – 103 = 896
Or (n – 1) = 896/7 = 128
Or n = 128 + 1 = 129
So, the total number of such terms is 129.

Sum of an AP = [(First term + Last term)/2] × number of terms


So, Sum of all the 3-digit numbers that leave a remainder of 5 when divided by 7 = [(103 + 999)/2] × 129 = 71079.
Hence, option (a) is correct.

 12 www.visionias.in ©Vision IAS


.

42 (c)
36 km/hr = 36 × 5/18 m/sec = 10 m/sec

45 km/hr = 45 × 5/18 m/sec = 12.5 m/sec
So, their relative speed = 10 + 12.5 = 22.5 m/sec
Time taken by them to meet each other = Length of the bridge / Relative speed = 315/22.5 = 14 seconds
Hence, option (c) is correct.

43 (d)
Let X be the passing marks and Y be the maximum possible marks.

According to the question, Ravi scored 37.5% marks in the annual exam and failed by 68 marks.
So, X = [(Y × 37.5)/100] + 68 ……(i)

And, his sister got 47.25% marks, which were 49 marks more than the passing marks.
So, X = [(Y × 47.25)/100] – 49 ……(ii)

By subtracting equation (i) from (ii), we get:


[(Y × 47.25)/100] – [(Y × 37.5)/100] = 68 + 49
Or Y (47.25 – 37.5)/100 = 117
Or Y = (100/9.75) × 117
Or Y = 1200
Hence, option (d) is the correct answer.

44 (a)
‘REPLACE’ is coded as ‘ECAOPER’

O is the opposite letter of L in the English alphabet.

Similarly,

Y is the opposite letter of B in the English alphabet.


Hence, option (a) is the correct answer.

 13 www.visionias.in ©Vision IAS


.

45 (c)

46 (d)
Speed of P = 54 km/hr = 54 × (5/18) = 15 m/sec
Speed of Q = 36 km/hr = 36 × (5/18) = 10 m/sec
Their relative speed = 15 – 10 = 5 m/sec
It means that, every second P takes a lead of 5 m over Q.
Therefore, the time after which they would meet = 400/5 = 80 seconds.
Hence, option (d) is correct.

47 (a)
Let the amount of money earned by one man and one woman in 1 day be ‘a’ and ‘b’ respectively.

3 men and 3 women earn Rs. 31200 in 8 days.


So, 8 × (3a + 3b) = 31200
Or 8 × 3(a + b) = 31200
Or a + b = 31200 / (3 × 8) = 1300
Or a + b = Rs. 1300 ……………. (i)

Also, 8 men and 12 women earn Rs. 64000 in 5 days.


So, 5 × (8a + 12b) = 64000
Or 5 × 4 × (2a + 3b) = 64000
Or 2a + 3b = 64000 / (5 × 4) = 3200
Or 2a + 3b = 3200

Solving equations (i) and (ii), we get:


a = Rs. 700 and b = Rs. 600

So, the money that 9 men and 5 women can earn in 7 days = [9 × 700 + 5 × 600] × 7 = Rs. 65,100
Hence, option (a) is correct.

48 (c)
According to question, when one tree is planted in each house, three trees would be left, but when two trees are
planted in each house, three houses would be left without any tree. Let’s solve the question with the help of options
and check if they satisfy this condition:

 14 www.visionias.in ©Vision IAS


.

Option (a): When 1 tree is planted in each house, 3 trees would be left, but when 2 trees are planted in each house,
no house is left. So, this statement doesn’t satisfy the condition.
 Option (b): When 1 tree is planted in each house 4 trees would be left. So, this statement doesn’t satisfy the
condition.
Option (d): When 1 tree is planted in each house 3 trees would be left, but when 2 trees are planted in each house
2 houses would be without any tree. So, this statement doesn’t satisfy the condition.
Option (c): When 1 tree is planted in each house 3 trees would be left, but when 2 trees are planted in each house
3 houses would be left without any tree. Hence, this statement satisfies the condition.

49 (a)
Statement 1 is correct. The passage states, “The United Nations Intergovernmental Panel on Climate Change
report has warned that unless governments everywhere reassess their energy policies, the world will become
uninhabitable”. The author does not leave any room for ambiguity when he quotes that the world will become
unlivable if the governments do not revisit their energy policies. Hence, it is a correct statement.
Statement 2 is incorrect. The passage states, “A critical question is whether African governments are doing enough
to support citizens in combating climate change effects”. It only asks a question and does not answer whether
African governments are doing a good job or not. Hence, it is not a correct statement.

50 (d)
Statement 1 is incorrect. The passage mentions, “The global demand for fresh water will exceed supply by 40 per
cent by 2030”. It means that demand will exceed supply by 40% in 2030. That is, if the demand is 140 litres, the
supply would be 100 litres only. It does not mean that the demand for fresh water will become 40% higher than the
present level. Hence, it is not a correct statement.
Statement 2 is incorrect. Refer to the line: “Around 97 per cent of the usable freshwater comes from groundwater
and the reliance on groundwater is immense.” It is true that almost all freshwater requirement is currently met by
ground water. Yet, we cannot conclusively say that groundwater is the only source of freshwater. There could be
other sources which are relatively untapped. Hence, it is an incorrect statement.

51 (d)
We can label the given figure for our convenience as follows:

The triangles in the above figure are:


ΔDEC, ΔAGD, ΔBHE, ΔDFE, ΔDGF, ΔHFE, ΔHIB, ΔAGJ, ΔJKI.
Also, ΔDCB, ΔACE, ΔADF, ΔBFE, ΔAJD, ΔBIE, ΔAFB, ΔABD, ΔABE, ΔDKE, ΔBDE, ΔADE, ΔABC.
Hence, option (d) is the correct answer.

52 (b)
Total dresses in showroom = 500
Total number of blue & white dresses = 0.78 x 500 = 390

 15 www.visionias.in ©Vision IAS


.

Now, white dresses are 5% lesser in number than blue dresses. Let number of blue dresses be x. So, Number of
white dresses = 0.95x
 Hence 0.95x + x = 390, i.e. x = 200
Hence number of blue dresses = 200
And number of white dresses = 0.95 x 200 = 190
Now, Number of red dresses = 50% of white dresses = 0.5 x 190 = 95
Remaining 15 must be dresses of other colours.
Hence, option (b) is the correct answer.

53 (b)
Let, x, y, z be the three co-prime numbers.
According to the question,
xy = 459 and yz = 621
Since, y is common between xy and yz.
So, y = H.C.F of 459, 621 = 27
Therefore, x = 459/y = 459/27 = 17
And z = 621/y = 621/27 = 23
Sum of these three co-prime numbers = x + y + z = 17 + 27 + 23 = 67
Hence, option (b) is the correct answer.

54 (a)

From the figure, it is clear that 74 – 27 = 47 is the missing number.


Hence, option (a) is the correct answer.

55 (d)
If all three integers x, y, and z are even, then their product must also be even.
So, statement 1 alone is sufficient to answer the question.

If the sum of x, y, and z is even, then following cases are possible:


* all of them are even. In such a scenario, their product must also be even.
* one is even and two are odd. In such a scenario, their product must also be even.
Therefore, either way the product of x, y, and z would be even.
 16 www.visionias.in ©Vision IAS
.

So, Statement 2 alone is also sufficient to answer the question.
Therefore, the answer is (d), i.e. either statement alone is sufficient to answer the question.


56 (c)
Let the number of black, blue and red coloured balls in the box be 3x, 8x and 7x respectively.

According to the question,


7x – 3x = multiple of 12 and 9
The multiple of 12 and 9 must be in the form of 36k.
So, 7x – 3x = 36k
Or 4x = 36k
Or x = 36k/4 = 9k

For x to be a natural number and have the minimum possible value, k should be equal to 1.
So, x = 9 × 1 = 9
Thus, the total number of balls in the box = 3x + 8x + 7x = 18x = 18 × 9 = 162
Hence, option (c) is correct.

57 (b)
Let the age of Sania be X years and the age of her daughter be Y years.
Now according to the question,
Y=X-Y
Or 2Y = X
Given that, X = 30 years
So, Y = 15 years
Hence, present age of Sania’s daughter = 15 years
Therefore, age of Sania’s daughter 4 years ago = 15 - 4 = 11 years
Hence, option (b) is correct.

58 (b)
Statement 1 is correct. The passage mentions, “The worship of female deities associated with fertility and
childbirth, considered protectresses of children with the power to ward off disease..., such functions were associated
with yakshis”. It clearly means Yakshis were associated with fertility. Hence, it is a correct statement.
Statement 2 is incorrect. The passage does talk about the worship of serpents. But, nowhere does it mention that
these serpent worships were a part of Shaivism. Hence, it is not a correct statement.

59 (b)
Statement 1 is incorrect. Refer to the line, “He introduced a new doctrine known as Doi Moi, which meant
renewing the economy, as the Chinese had already begun to do, by moving towards the free market, in an attempt
to raise living standards to the level enjoyed by Vietnam's neighbours”. This clearly implies that economic reforms
were introduced in China. But they were not influenced by the Doi Moi initiative, as the author clearly says that

 17 www.visionias.in ©Vision IAS


.

Chinese reforms had already commenced by the time Doi Moi doctrine was introduced. Hence, it is not a correct
statement.
 Statement 2 is correct. The passage clearly states, “Vietnamese troops were withdrawn in September 1989 ... This
was a great relief for the regime, since it freed vast sums of revenue which could now be invested in the economy”.
The money that was spent on military expenditures could now be invested in economy - an economic relief for
Vietnam. Hence, it is a correct statement.

60 (a)
Assumption 1 is valid. The given assumption is correct as it is based on the line, “However, the truth is that human
behaviour is often the weakest link in the online security chain.” Here, we can say that even the best cybersecurity
solution may not ensure complete online safety for users, because it does not deal with the weakest link which is
human behaviour. So, in addition to technology, human behaviour needs to be in sync with the emerging trend of
cyberattacks to prevent fraud. Hence, the given assumption is correct.
Assumption 2 is invalid. The given statement is not correct because the biases of people can be changed by
themselves or through awareness programmes. The job of the cybersecurity experts is to develop technological
solutions, and not to engage in psychological domains. So, this assumption is not correct.

61 (d)
Inference 1 is incorrect. The passage nowhere discusses the context of schemes for the promotion and protection
of artisans, or bureaucratic machinery and corruption therein. It mostly talks about the features and challenges of
the Indian artisan community. Hence, this inference is beyond the scope of the passage.
Inference 2 is incorrect. The line, “At the higher end of the retail segment, top designers and labels have huge
margins, but the sector’s current supply chain structure is inequitable, with actual creators of products getting very
little” shows that huge margins are gained by top designers and labels. But inferring that government eliminating
(an extreme step) the traders will lead to a raise in the margins for artisans is not correct.

62 (c)
216 smaller cubes means the larger cube is composed of 6 × 6 × 6 blocks.
Total number of small cubes having only 2 faces painted = 12 × (n – 2) = 12 × (6 – 2) = 48
(n = 6 and 12 is the number of edges of a cube)
The 2 painted faces can be red-blue, red- green or blue-green.
So, the number of small cubes having the combination of only red and blue = 48/3 = 16
The number of cubes with 3 sides painted, i.e. having combination of red, blue & green will always be 8 (the corner
cubes).
‫ ׵‬Total number of small cubes having both red and blue colours on them = 16 + 8 = 24

63 (c)
From Statement 1:
Let two digits be x and y.
Multiplication of two digits is 18.
x×y = 18 …………………………eq(1)
Ratio of digits is 1: 2.
x/y = 1/2
֜ x = y/2
Putting the value of x in eq(1), we get:
 18 www.visionias.in ©Vision IAS
.

(y/2) ×y = 18
Or y2 = 18×2 = 36

Or y = 6
Now, x = y/2 = 6/2 = 3
Therefore, Statement 1 alone is sufficient to answer the Question.

From Statement 2:
Let two digits be b and a.
Sum of two digits is 9.
a+b=9 ………eq(1)
The units digit is half of the tens digit.
a/b = 2
֜ a = 2b ………eq(2)
Putting the value of a in eq(1), we get:
2b + b = 9
Or 3b = 9
Or b = 3
Now, a = 2b = 2 × 3 = 6
Therefore, Statement 2 alone is also sufficient to answer the Question.

Thus, either Statement-1 alone or Statement-2 alone is sufficient to answer the Question.
Hence, option (c) is the correct answer.

64 (c)
Let the number of matches lost by Neatherlands be x.
According to the question,
Total points received by winning games = Total points lost by losing games
Or (108 – x) × 1 = x × (1/3)
Or (x/3) + x = 108
Or 4x/3 = 108
Or 4x = 108 × 3
Or x = 81
Hence, Netherlands lost 81 matches.
Hence, option (c) is correct.

65 (c)
According to the question,
ଷ ଶ
ͷ ଶ and ͳͲ ହare integral multiples of the number.
ଷ ଶ
ͷ = 13/2 and ͳͲ = 52/5
ଶ ହ

 19 www.visionias.in ©Vision IAS


.


Required highest possible number = H.C.F of fractions = H.C.F of numerators/L.C.M of denominators = H.C.F of
 (13, 52) / L.C.M of (2, 5) = 13/10
Hence, option (c) is the correct answer.

66 (a)
Let the speed of boat in still water be x km/hr.
Speed of stream = 3 km/hr
Downstream speed = (x + 3) km/hr
Upstream speed = (x – 3) km/hr
Time taken in journey = 8 hrs.

According to the question,


[48/(x + 3)] + [48/(x – 3)] = 8
Or (x – 3 + x + 3)/(x2 – 9) = 8/48
Or 2x/(x2 – 9) = 1/6
Or x2 – 12x – 9 = 0

Roots of quadratic equation ax2 + bx + c = 0 can be found by using the formula [–b ± {b2 – 4ac}1/2] / 2a
So, Roots of the given quadratic equation = [–(–12) ± {122 – 4 × 1 × (-9)}1/2] / 2
= [12 ± (144 + 36)1/2] / 2
= [12 + (180)1/2] / 2 (Neglecting the negative value)
= (12 + 13.4)/2
= 12.7
So, the speed of boat in still water is 12.7 km/hr (approximately).
Hence, option (a) is correct.

67 (b)
Total number of balls = 6 + 5 + 4 = 15
Probability that both the balls drawn are of the same colour = [6C2 + 5C2 + 4C2] / 15C2 = [{(6 × 5)/2} + {(5 × 4)/2} +
{(4 × 3)/2}] / [(15 × 14)/2] = (15 + 10 + 6)/105 = 31/105

So, the probability that the two balls are of different colours = 1 - probability that both the balls are of the same
colour = 1 – (31/105) = 74/105
Hence, option (b) is correct.

68 (b)
A works twice as fast as B, who works thrice as fast as C.
So, A : B = 2 : 1, B : C = 3 : 1
Or A : B : C = (2 × 3 : 1 × 3 : 1) = 6 : 3 : 1
So, the ratio of the efficiencies of A, B and C is 6 : 3 : 1.

 20 www.visionias.in ©Vision IAS


.

Let efficiencies of A, B and C be 6x, 3x and x respectively.
Work done in 1 day by all of them working together = 6x + 3x + x = 10x


Total Work = Rate of Work × Time Taken = 10x × 15 = 150x


80% of the total work = 150x × 80/100 = 120x

So, time taken by B alone to finish 80% of the task = 80% of the total work/efficiency of B = 120x/3x = 40 days
Hence, option (b) is correct.

69 (d)
According to the question,
(16/20) + (x/30) = 1
Or x/30 = 1 – (16/20) = 4/20 = 1/5
Or x = (1/5) × 30 = 6 minutes
Hence, option (d) is correct.

70 (a)
Let tens digit of the number be a and unit digit of the number be b.
Then, Original number = 10a + b
When this two-digit number is multiplied by the difference of its digits, the product is 558.
So, (10a + b) × (a – b) = 558 ………….. (i)

When the digits are interchanged, the new number = 10b + a


When the reverse number is multiplied by the difference of its digits, the result is 234.
So, (10b + a) × (a – b) = 234 …………….. (ii)

On performing the operation Equation (i) – Equation (ii), we get:


[(10a + b) × (a – b)] – [(10b + a) × (a – b)] = 558 – 234 = 324
Or (a – b) [10a + b – (10b + a)] = 324
Or (a – b) (9a – 9b) = 324
Or 9(a – b) (a – b) = 324
Or (a – b)2 = 324/9 = 36
Or (a – b) = 6 (neglecting the negative sign)
So, the difference between the digits of the given number is 6.
Hence, option (a) is correct.

71 (d)
Let the marked selling price of an object be x.
Selling price during the season sale = x – 20% of x = 0.8x

 21 www.visionias.in ©Vision IAS


.

Selling price after the end of season sale = x - 10% of x = 0.9x
Increase in selling price = 0.9x – 0.8x = 0.1x

Percentage increase in selling price = (0.1x/0.8x) × 100 = 12.5%
Hence, option (d) is the correct answer.

72 (d)
It's given that a > b and c > 0.
1. a - b > c
This need not be true, if say a = 20, b = 18, and c = 5.

2. a + b > c
This need not be true, if say a = 3, b = 2, and c = 8.

3. ac > b
This need not be true, if say a = -3, b = -4, and c = 2.
Hence, option (d) is the correct answer.

73 (b)
As it’s evident that there are four o and 3 x in the 3×3 check box, it’s the turn of the player using the x symbol.

If the player using x crosses the check box 1, then the player using o will fill in o in the check box 2 and will emerge
as the winner (as there will be three ‘o’s diagonally). However, if the player using x crosses the check box 2, then
the player using o will fill in o in the check box 1 and the game will not produce any winner (i.e. it will be a draw).
So, the probability that the game will not produce any winner from the present situation = 1/2
Hence, option (b) is the correct answer.

74 (c)
Let the total work be w units.
Total time taken by Samar to complete the work = D days
Now, Ankit does 0.25w units of work in 1.25D days.
Hence, Ankit will take 5D days to complete the total w units of work.
Efficiency of Samar = 1/D
Efficiency of Ankit = 1/5D
Hence, the required ratio of efficiency of Ankit to that of Samar = 1/5D : 1/D = 1 : 5
Hence, option (c) is the correct answer.

 22 www.visionias.in ©Vision IAS


.

75 (a)
Inference 1 is correct. The given inference is correct because of the lines, “For example, one theory of justice
 suggests that we should help those who are unable to benefit from the virtues of the “genetic lottery …” and “A
contrasting theory, however, says that we are entitled to whatever we earn, and we don’t owe anything to anyone
else.” These lines show two different and contrasting approaches to helping people in need (altruism), but do not
give a clear answer about which theory to follow. Therefore, it would be correct to infer that philosophy of justice
does not give a clear answer regarding altruistic aspects of life.
Inference 2 is incorrect. The line, “For example, one theory of justice suggests that we should help those who are
unable to benefit from the virtues of the “genetic lottery”—intelligence, health, aptitudes, and even the capacity to
do hard work” shows that even the capacity to do hard work is decided by genetics. Therefore, to infer that one
should believe only in hard work to achieve success would not be correct.

76 (d)
Statement 1 is incorrect. The Indus Water Treaty (IWT) governs six rivers that flow across the two countries. The
passage does not mention that all the rivers that flow between the two countries are governed by the treaty. It is
evident from the line, “…the Indus Waters Treaty, which governs the sharing of six rivers flowing through the two
countries”. Hence, it is not a correct statement.
Statement 2 is incorrect. According to the passage, “Calls for amending or renegotiating the Indus Waters Treaty
are being made for more than two decades now, on both sides”. It does not mention that Pakistan has been more
vocal in supporting the amendment to the Indus Water Treaty. Hence, it is not a correct statement.

77 (a)
Statement 1 is correct. Refer to the line, “It can be viewed as India’s countermeasure to Pakistan’s repeated
objections to every irrigation or power project on the Indian side of the Indus basin.” The demand to amend the
treaty may be driven by bilateral issues between the two countries. Hence, it is a correct statement.
Statement 2 is incorrect. Refer to the line, “…Pakistan’s repeated objections to every irrigation or power project
on the Indian side of the Indus basin”. Pakistan’s resistance is to both power and irrigation projects. It is not limited
to power projects only. Secondly, the subject of solar and wind energy has not been covered in the passage.
Therefore, this answer option is incorrect.

78 (d)
Statement 1 is incorrect. Refer to the line, “…led many to proclaim that Google’s days as the numero uno search
engine are numbered. While that’s debatable…” When ChatGPT was introduced, there were speculations that it
will replace Google’s search engine, but it is yet to be conclusively proven. Hence, it is not a correct statement.
Statement 2 is incorrect. According to the passage, “However, even as the idea to include chatbot in search is
new, Google will not be the first to do so when it does.” This implies that Google is not the first firm to integrate a
chatbot in its search engine. Hence, it is not a correct statement.

79 (a)
Statement 1 is correct. The passage says, “Where Google serves up a list of internet links in response to your
queries, ChatGPT goes the extra mile in providing information in clear, simple sentences that are much easier to
understand. Add to that the ability to generate content from scratch”. This clearly means that ChatGPT offers
information that is easier to comprehend. Hence, it is a correct statement.
Statement 2 is incorrect. Refer to the line, “The company (Google) reportedly declared a “code red” fearing that
the AI chatbot could disrupt its search business.” There is no doubt that Google is affected. But we do not have
sufficient information in the passage to prove that Google is affected the most in tech industry. The author has
mentioned only two companies – Google and Microsoft.

 23 www.visionias.in ©Vision IAS


.

80 (b)
Method I: (solving with the help of options)

It is given that total number of students in class 8th is 12 times of the number of rows formed.
So, total number of students must be divisible by 12.
Only options (b) and (d) are divisible by 12.
Now, if total number of students is 60, then number of rows will be 5; and if total number of students is 108, then
number of rows will be 9.

Case 1: When the number of rows is 9


It is given that first row contains 6 students and each subsequent row contains three more students than the preceding
row.
Therefore, total number of students = 6 + 9 + 12 +15 +18 +21 + 24 +27 + 30 = 162 students
As mentioned in option (d), only 108 students (12x9) have to be accommodated in 9 rows. There is a contradiction.
Hence, option (d) is not the correct answer.

Case 2: When the number of rows is 5


Total number of students = 6 + 9 + 12 +15 +18 = 60 students
It satisfies the condition of the question.

So, total number of students is 60.


Hence, option (b) is the correct answer.

Method II:
Let there be n rows.
It is mentioned that total number of students is 12 times the number of rows = 12n
Also, it is given that the first row contains 6 students, and each subsequent row contains three more students than
the preceding row. Hence, number of students in each row will be like 6, 9, 12, 15 and so on….
This series forms an arithmetic progression (AP), whose first term (a) is 6 and common difference (d) is 3.

We know that in an arithmetic progression,


Sum of n terms = (n/2) [2a + (n-1) d]
Or 12 n = (n/2) [2 × 6 + (n-1)3]
Or 24 = 12 + 3n - 3
Or 3n = 24 – 12 + 3 = 15
Or n = 5
Hence, number of rows = 5
Now, total number of students = 12n = 5 × 12 = 60
Hence, option (b) is the correct answer.

 24 www.visionias.in ©Vision IAS

You might also like